LSAT and Law School Admissions Forum

Get expert LSAT preparation and law school admissions advice from PowerScore Test Preparation.

 Administrator
PowerScore Staff
  • PowerScore Staff
  • Posts: 8916
  • Joined: Feb 02, 2011
|
#85355
Complete Question Explanation

Cannot Be True—Numbers and Percentages. The correct answer choice is (A)

Let us first review the facts given in the stimulus:

          Statement: For a ten‑month period, the total monthly sales of new cars within the country of
          Calistan remained constant.

     The wording of this statement should alert you that numbers and percentages in the form of
     market share (“monthly sales...remained constant”) may be an issue in this problem. Note
     that the statement is a simple fact; no explanation is given for why the total sales stayed
     constant.

          Statement: During this period the monthly sales of new cars manufactured by Marvel
          Automobile Company doubled, and its share of the new car market within
          Calistan increased correspondingly.

If total monthly sales of new cars remains constant and Marvel’s sales doubled, then Marvel’s
share of the new car market must also have doubled. Again, no explanation for Marvel’s
increase is given; the increase is just stated as a fact.

          Statement: At the end of this period, emission standards were imposed on new cars sold
          within Calistan.

     Imposing new emission standards serves as a chronology marker in this stimulus. Again, no
     explanation is given for why the new standards were imposed.

          Statement: During the three months following this imposition, Marvel Automobile
          Company’s share of the Calistan market declined substantially even though its
          monthly sales within Calistan remained constant at the level reached in the last
          month of the ten‑month period

     This sentence is the key to the stimulus. From a numbers and percentages standpoint, we are
     given two pieces of related information: during the three months after the emissions standards
     were imposed, Marvel’s monthly sales of new cars within Calistan remained constant at
     the pre-standards level, and at the same time Marvel’s share of the market declined. From
     the discussion in Chapter Seventeen, we know that if sales remain constant but the share
     represented by those sales decreased, then the overall sales in the market must have increased.
     For example:
Capture.PNG

     The other important part of this sentence is what is not said. No cause is given for Marvel’s
     decline, and you cannot assume that the new emissions standards are the cause of the decline
     (causal indicators are needed to convey causality, and none are present in this stimulus).
     Remember, one error of causal reasoning is to assume that because two things occur in
     sequence that one caused the other. There could be many different explanations for Marvel’s
     decline other than the new emission standards. For example, Marvel could have raised their
     car prices or perhaps Marvel received some negative publicity about the quality of their
     cars. Regardless, the problem is clearly designed to test whether you will fall into the trap
     of assuming that the new emission standards caused Marvel to lose market share, so read
     carefully and do not fill in the “spaces” in the stimulus.

Answer choice (A): This is the correct answer. As shown in the discussion of the last sentence of the
stimulus, in the three months after the imposition of the emissions standards, the total monthly car
sales in Calistan must have risen, and since Marvel’s monthly car sales remained constant, we can
conclude that the sales of other car makers must have risen. Since this answer claims they decreased,
this answer cannot be true and is correct.

Answer choice (B): This answer is possibly true. The stimulus indicates that Marvel doubled sales
and market share in the ten months prior to the imposition of the emissions standards, and it is
possible that in the three months prior to the implementation of the new standards the market share
of the other companies decreased.

Some students look at this answer and assume that it must be true based on the first two sentences
of the stimulus. But that judgment assumes that Marvel’s growth during the ten-month period was
constant, a circumstance never stated by the author. It would be consistent with the stimulus if
Marvel doubled sales in the first month and then remained constant for the remaining nine months,
which would allow for its competitors to gain market share during the last three months.

Answer choice (C): No reason is given for Marvel’s loss of market share (or alternately, the increased
sales of other manufacturers), so it is possible that the new emission standards actually decreased
Marvel’s loss of market share (or alternately, the emission standards limited the increase in sales of the
other manufacturers). Remember, no explanation is given for the situation after the imposition of the
emission standards, so whatever happened in the absence of the standards could always be true.

Answer choice (D): This answer could occur because Calistan’s future car sales could fall due to a
variety of causes (including the emission standards). This is true regardless of whether the emissions
standards remain in force because we know nothing of the effect of the standards.

Answer choice (E): No information is given about profit in the stimulus, so this answer choice could
be true.
You do not have the required permissions to view the files attached to this post.
 khana87
  • Posts: 11
  • Joined: Dec 24, 2011
|
#4766
I don't see how "a" is the answer. I thought the decrease in monthly sales of cars by other companies following the imposition was plausible since they might not meet the emission standards. I thought that "e", where marvel's average profit on each car sold increased after the imposition, would be untrue because their share declined substantially after the imposition. Not sure if I'm grasping the stimulus properly. :-?
 Steve Stein
PowerScore Staff
  • PowerScore Staff
  • Posts: 1153
  • Joined: Apr 11, 2011
|
#4779
The emissions standards were imposed on all new cars prior to the three-month period in question, so we can presume that all cars made during that three-month period met these new standards.

Marvel's sales remained constant, yet their market share decreased. This means that total sales in Calistan had to have increased--this is the only way to maintain sales but drive down market share.

Additionally, if Marvel's market share decreased, the market share of the competition had to have increased correspondingly.

Thus, there is no way that the competition could have experienced a decrease as well.

Some of these numbers and percentage questions can be tricky! let me know whether this is clear--thanks!

~Steve
 Oscarg104
  • Posts: 9
  • Joined: Aug 24, 2018
|
#50027
Hi, I still do not understand how A is the best possible answer here. I picked D, however, I was not exactly confident in picking that as my answer, neither. Can you please explain thoroughly as to why each answer choice is wrong, as well as why the correct answer choice is right. Thanks!
 Adam Tyson
PowerScore Staff
  • PowerScore Staff
  • Posts: 5153
  • Joined: Apr 14, 2011
|
#50053
You bet, Oscarg104! Let's try thinking about the numbers here, and we'll keep them simple. First, at the beginning of the 10 month period, let's say Marvel sold 5 cars, and that was 5% of the market - that means 100 total cars were sold per month. At the end of the period, then, Marvel was selling 10 cars, but the market stayed the same, just 100 cars, and Marvel's share went up to 10%, just as the stimulus told us (their sales doubled and so did their share of the market). So that's where things stand at the end of those 10 months - 100 cars per month, with Marvel selling 10 of them, which is 10%. So far, so good?

Now for the next 3 months, Marvel's sales remained constant - 10 cars per month - but it's share of the market declined substantially. So maybe it went from 10% of the market back down to 5%? For those 10 cars to equal 5% of the market, the market must have doubled to 200 cars per month. That has to be true in order for these numbers and percentages to make sense. And those extra 100 cars must have been sold by other companies, because Marvel stayed steady at 10 per month.

So, what cannot be true? It cannot be true that the total market stayed the same or declined. That's answer A - it cannot be that sales by other companies went down. Boom, winner!

Answer B could be true - if Marvel's share was increasing from 5% to 10%, then the combined share of the other companies went from 95% down to 90%, and some of that decrease could have occurred in those last three months of the 10 month period before the new emissions standards hit.

Answer C could be true, because we cannot know what would have happened in an alternate reality where the standards were not changed. Basically, ANYTHING could be true in that hypothetical situation! We cannot disprove it, so it could be true.

Answer D is the same as answer C in that regard. There is no way to know what will happen in the future if something does or does not happen. Since we cannot disprove answer D, it could be true. We just can't know!

Answer E could also be true, because we know nothing about profits. They could be up, down, or the same - there is no evidence anywhere about profits, so we cannot disprove this, and it could be true.

The big theme here is that if we cannot disprove it, it could be true. Not that it must, not even that it probably will be, but simply that it cannot be ruled out. For a Cannot Be True question (which are fairly rare in LR, but common in LG), we have to pick the one answer that is absolutely, certainly not possible. Anything unknown is a wrong answer!

I hope my little illustration with the numbers helps! Let us know if you need any more help!
 Oscarg104
  • Posts: 9
  • Joined: Aug 24, 2018
|
#50502
That was awesome, Adam! Thanks!
 AM4747
  • Posts: 17
  • Joined: Oct 22, 2018
|
#61523
Hello,

I completely understand the reasoning behind the correct answer (A). However, there seems to be something lacking, which is the COMBINED monthly sales of other companies. In that case the decrease of monthly sales would be impossible according to the information given in the stimulus. But the way in which answer choice A is phrased leaves room for one to interpret that perhaps only one company had a decrease whereas other companies had an increase in their monthly sales thereby allowing for the total cars sold to increase, in turn explaining the decrease of Marvel in market share.

For this answer to be correct, therefore, either there must be only one company other than Marvel, or the answer choice must state combined.

Any and all help would be appreciated,
Happy new year!
User avatar
 Dave Killoran
PowerScore Staff
  • PowerScore Staff
  • Posts: 5853
  • Joined: Mar 25, 2011
|
#61540
AM4747 wrote:Hello,

I completely understand the reasoning behind the correct answer (A). However, there seems to be something lacking, which is the COMBINED monthly sales of other companies. In that case the decrease of monthly sales would be impossible according to the information given in the stimulus. But the way in which answer choice A is phrased leaves room for one to interpret that perhaps only one company had a decrease whereas other companies had an increase in their monthly sales thereby allowing for the total cars sold to increase, in turn explaining the decrease of Marvel in market share.

For this answer to be correct, therefore, either there must be only one company other than Marvel, or the answer choice must state combined.

Any and all help would be appreciated,
Happy new year!
Hi AM4747,

Thanks for the question! I'm not following your reasoning here. Answer choice (A) clearly states that "total monthly sales within Calistan of new cars by companies other than Marvel Automobile Company decreased," which reflects the combined numbers. Since a combination that results in a decrease would be a problem, this is stated properly. Whatever happens inside those other car maker numbers is irrelevant (at least in the way this question is put together).

If I've missed something here in your explanation, please let me know, but overall there's no problem with (A).

Thanks!
 kupwarriors9
  • Posts: 73
  • Joined: Jul 01, 2021
|
#88912
What is the difference between A and B for #1? Thanks


(P.S. I'm a science major with no. familiarity whatsoever of business or business terms please help me!!)
User avatar
 Stephanie Oswalt
PowerScore Staff
  • PowerScore Staff
  • Posts: 811
  • Joined: Jan 11, 2016
|
#88921
kupwarriors9 wrote: Tue Jul 20, 2021 9:32 am What is the difference between A and B for #1? Thanks


(P.S. I'm a science major with no. familiarity whatsoever of business or business terms please help me!!)
Hi Kup!

I've moved your question to the thread discussing this topic. Please review the above explanation and discussion, and let us know if that helps! Thanks!

Get the most out of your LSAT Prep Plus subscription.

Analyze and track your performance with our Testing and Analytics Package.